Tải bản đầy đủ (.pdf) (5 trang)

Bất đẳng thức diện tích tam giác

Bạn đang xem bản rút gọn của tài liệu. Xem và tải ngay bản đầy đủ của tài liệu tại đây (142.31 KB, 5 trang )

Một bất đẳng thức hay về diện tích tam giác
Trần Lê Bách
10A1 Trường THPC Chuyên Lê Quý Đôn
Đà Nẵng, Việt Nam

1

Định lý cơ sở
Chúng ta xét định lý quen thuộc sau:

Định lý 1.1. Cho m, n, p là các số thực thỏa mãn m + n, n + p, p + m và mn + np + pm là các số không
âm. Đặt a, b, c là độ dài 3 cạnh của tam giác ABC. Khi đó

ma2 + nb2 + pc2 ≥ 4 mn + np + pm.S
với S là diện tích của tam giác ABC
Chứng minh. Theo Định lý hàm số cosines, ta có

ma2 + nb2 + pc2 ≥ 4 mn + np + pm.S

⇔ma2 + nb2 + p(a2 + b2 − 2ab.cosC) ≥ 2ab.sinC mn + np + pm

a
b

(m + p) + (n + p) ≥ 2 mn + np + pm.sinC + p.cosC
b
a
Sử dụng Bất đẳng thức Cauchy-Schwarz ta suy ra

2
mn + np + pm.sinC + p.cosC ≤ p2 + mn + np + pm



sin2 C + cos2 C

= (m + p)(n + p)
Mặt khác
(m + p)
Do đó

b
a
+ (n + p)
b
a

2

≥ 4(m + p)(n + p)


ma2 + nb2 + pc2 ≥ 4 mn + np + pm.S

Nhận xét 1.1. Một câu hỏi quan trọng được đặt ra là đẳng thức xảy ra khi nào? Từ Bất đẳng thức
Cauchy-Schwarz ta có đẳng thức xảy ra khi và chỉ khi
(m + p) ab = (n + p) ab
cosC
sinC
= √mn+np+pq
p
√a
p+n

cos2 C
p2



=
=

√ b
m+p
sin2 C
mn+np+pq

=

1
(m+p)(n+p)

Thay cosC, b tương ứng vào biểu thức: c2 = a2 + b2 − 2ab.cosC ta thu được
c2 = a2 + a2
2

m+p
n+p

m+p
p
=1+
−2
n+p

n+p

c
m+n
⇔ = √
a
n+p
a
c
⇔√
=√
n+p
m+n


c
a

m+p
− 2a2
n+p

1

p
(m + p)(n + p)


Tương tự ta có đẳng thức xảy ra khi và chỉ khi



2

a
b
c
.
=√
=√
n+p
m+p
m+n

Một số ứng dụng

Đặt a, b, c là độ dài 3 cạnh của tam giác ABC và S là diện tích của tam giác đó. Ta có một số bất đẳng
thức sau:

Bài 2.1. (Bất đẳng thức Hadwiger-Finsler)

a2 + b2 + c2 ≥ 4 3.S + (a − b)2 + (b − c)2 + (c − a)2
Chứng minh. Bất đẳng thức đã cho tương đương với
a2


b+c−a
c+a−b
a+b−c
+ b2
+ a2

≥ 4 3.S
a
b
c

Theo Định lý 2.1, ta chỉ cần chứng minh

cyc

(b + c − a)(c + a − b)
≥3
ab

Hay
a(a − b)(a − c) + b(b − c)(b − a) + c(c − a)(c − b) ≥ 0
Đây là Bất đẳng thức Schur bậc ba.

Bài 2.2. (Trần Lê Bách)

3
4
a2 b + b2 c + c2 a ≥ 8 27.S 2
Chứng minh. Từ Bất đẳng thức Hadwiger-Finsler, ta dễ dàng suy ra

ab + bc + ca ≥ 4 3.S
Bây giờ áp dụng Định lý 2.1, ta có


3
4

a2 b + b2 c + c2 a ≥ 4 ab + bc + ca.S ≥ 8 27.S 2
Đó chính là đpcm.

Bài 2.3. (Trần Lê Bách)
3abc ≥ 4 a2 + b2 + c2 .S
Cách một. Bất đẳng thức đã cho tương đương
a2

bc
ca
ab
+ b2 + c2
≥4
a
b
c

a2 + b2 + c2 .S

Theo Định lý 2.1, ta có
3abc ≥ 4 a2 + b2 + c2 .S
đpcm.
2


Cách hai. Bất đẳng thức đã cho có thể viết dưới dạng 9R2 ≥ a2 + b2 + c2 hay
sin2 A + sin2 B + sin2 C ≤

9
4


Phần còn lại xin dành cho bạn đọc.

Bài 2.4. (Trần Lê Bách)

3
4
(b + c − a)a2 + (c + a − b)b2 + (a + b − c)c2 ≥ 8 3.S 2
Chứng minh. Theo Định lý 2.1, ta có
(b + c − a)a2 + (c + a − b)b2 + (a + b − c)c2 ≥ 4

2ab + 2bc + 2ca − a2 − b2 − c2 .S

Từ Bất đẳng thức Hadwiger-Finsler, ta có
4

2ab + 2bc + 2ca − a2 − b2 − c2 .S ≥ 4



3
4
4 3.S.S = 8 3.S 2 đpcm.

Bài 2.5. Chứng minh bất đẳng thức sau với x, y, z là các số thực dương.

x 2
y 2
z
a +

b +
c2 ≥ 2 3.S
y+z
z+x
x+y
Chứng minh. Trước tiên ta sẽ chứng minh
yz
xz
3
xy
+
+

(y + z)(y + z) (y + x)(z + x) (x + y)(z + y)
4
Hay
x(y − z)2 + y(z − x)2 + z(x − y)2 ≥ 0
Bây giờ áp dụng Định lý 2.1 ta có
y 2
z
x 2
a +
b +
c2 ≥ 4
y+z
z+x
x+y
Do đó

xy

yz
xz
+
+
.S
(y + z)(y + z) (y + x)(z + x) (x + y)(z + y)


x 2
y 2
z
a +
b +
c2 ≥ 2 3.S
y+z
z+x
x+y

Bài 2.6. (Trần Lê Bách)
Chứng minh bất đẳng thức sau với mọi tam giác ABC không cân thỏa mãn 0 < a < b < c
abc


a−b
b−c
c−a
+
+
≥ 4 2.S
(b − c)(c − a) (c − a)(a − b) (a − b)(b − c)


Chứng minh. Ta có thẻ viết bất đẳng thức đã cho dưới dạng

bc(b − c)
ca(c − a)
ab
a2 +
b2 +
c2 ≥ 4 2.S
a(c − a)(a − b)
b(a − b)(b − c)
c(b − c)(c − a)
Từ hệ thức quen thuộc
bc
ca
ab
+
+
=1
(a − b)(a − c) (b − c)(b − a) (c − a)(c − b)
3



a2 + b2 + c2 ≥ ab + bc + ca
Ta suy ra
b2
c2
a2
+

+
≥2
(b − c)2
(c − a)2
(a − b)2
Theo Định lý 2.1 ta có đpcm.
Một bài toán tương tự:

Bài 2.7. (Trần Lê Bách)
Chứng minh bất đẳng thức sau với mọi tam giác ABC không cân thỏa mãn 0 < a < b < c
ab(a − b)
(b − c)(c − a)

2

+

bc(b − c)
(c − a)(a − b)

2

+

ca(c − a)
(a − b)(b − c)

2

> 4S


Bài 2.8. (Bất đẳng thức Pedoe)
Đặt a1 , b1 , c1 là độ dài các cạnh của tam giác A1 B1 C1 với diện tích S1 . Đặt a2 , b2 , c2 là độ dài các cạnh của
tam giác A2 B2 C2 với diện tích S2 . CHứng minh rằng
a21 (a22 + b22 − c22 ) + b21 (b22 + c22 − a22 ) + c21 (c22 + a22 − b22 ) ≥ 16S1 .S2
Cách một. Từ Định lý 2.1 ta có
a21 (a22 + b22 − c22 ) + b21 (b22 + c22 − a22 ) + c21 (c22 + a22 − b22 ) ≥
≥4

(a22 + b22 − c22 )(b22 + c22 − a22 ) + (b22 + c22 − a22 )(c22 + a22 − b22 ) + (c22 + a22 − b22 )(a22 + b22 − c22 ).S1

Mặt khác
(a22 + b22 − c22 )(b22 + c22 − a22 ) + (b22 + c22 − a22 )(c22 + a22 − b22 ) + (c22 + a22 − b22 )(a22 + b22 − c22 )
= (a2 + b2 − c2 )(c2 + a2 − b2 )(b2 + a2 − c2 )(a2 + b2 + c2 )
2

= 16 (S2 )
Do đó

a21 (a22 + b22 − c22 ) + b21 (b22 + c22 − a22 ) + c21 (c22 + a22 − b22 ) ≥ 16S1 .S2
Đó chính là đpcm.
Cách hai. Ta có
4cotA =

b2 + c2 − a2
2bc.cosA
=
1
S
2 bc.sinA


Bất đẳng thức đã cho tương đương
cotA2 .a21 + cotB2 .b21 + cotC2 .c21 ≥ 4S1
Mà cotA2 cotB2 + cotB2 cotC2 + cotC2 cotA2 = 1 nên theo Định lý 2.1 ta có đpcm.

Nhận xét 2.1. Bạn hãy thử tìm trường hợp đẳng thức xảy ra trong các bài toán trên?

4


3

Glossary

Định lý hàm số cosines
Với mọi tam giác ABC ta có.
BC 2 = AB 2 + BC 2 − 2AB.BC.cosα

Bất đẳng thức Schur
Cho a, b, c là các số không âm và r ≥ −1. Khi đó
ar (a − b)(a − c) + br (b − c)(b − a) + cr (c − a)(c − b) ≥ 0

Tài liệu
[1] Hojoo Lee: Topics in inequalities
[2] Vasile Cirtoaje: Algebraic inequalities: Old and New methods, Gil Publishing House
[3] Mathematics and Youth magazine, Viet Nam

5




×